Difference between revisions of "2016 AMC 12A Problems/Problem 3"

(Solution)
(Redirect to matching 10A problem)
 
(4 intermediate revisions by one other user not shown)
Line 1: Line 1:
==Solution==
+
#REDIRECT [[2016 AMC 10A Problems/Problem 4]]
<cmath>\text{rem}\left(\frac{3}{8},-\frac{2}{5}\right)</cmath>
 
<cmath>=\frac{3}{8}-\left(-\frac{2}{5}\right)\lfloor\frac{\frac{3}{8}}{-\frac{2}{5}}\rfloor</cmath>
 
<cmath>=\frac{3}{8}+\left(\frac{2}{5}\right)\lfloor -\frac{15}{16}\rfloor</cmath>
 
<cmath>=\frac{3}{8}+\left(\frac{2}{5}\right)\left(-1\right)</cmath>
 
<cmath>=\frac{3}{8}-\frac{2}{5}</cmath>
 
<cmath>=\boxed{\textbf{(B)}-\frac{1}{40}}</cmath>
 

Latest revision as of 12:51, 4 February 2016